Question

Given the information below please choose the correct answer with an explanation of why you chosed...

Given the information below please choose the correct answer with an explanation of why you chosed that answer.

Case 2 – New data set: ABP = 90/60 mm Hg, HR 145 Beats/min, cardiac output = 3 l/min, CVP = 12 mm Hg, PAP = 45/30 mm Hg, PCWP = 18 mm Hg, Ejection Fraction = 20%.

Mean arterial pressure (MAP)

                            70                mmHg

Systemic vascular resistance (SVR)

                                1547          dynes  

Mean pulmonary artery pressure (PA mean)

                                 17          mm Hg

Pulmonary vascular resistance (PVR)

                          453                  dynes

Stroke volume (SV)

                        21                      ml

***Regarding Case 2. The cardiac output is most likely lower than normal because:

preload is low

afterload is low

contractility is low

tachycardia is preventing adequate time during diastole to allow adequate filling of the ventricles

0 0
Add a comment Improve this question Transcribed image text
Answer #1

Answer: The cardiac output is most likely lower than normal because tachycardia is preventing adequate time during diastole to allow adequate filling of the ventricles.

This is because increase in the heart rate causes more blood flow and hence decrease in heart rate lowers the cardiac output.

Add a comment
Know the answer?
Add Answer to:
Given the information below please choose the correct answer with an explanation of why you chosed...
Your Answer:

Post as a guest

Your Name:

What's your source?

Earn Coins

Coins can be redeemed for fabulous gifts.

Not the answer you're looking for? Ask your own homework help question. Our experts will answer your question WITHIN MINUTES for Free.
Similar Homework Help Questions
  • Case 1 – New data set: ABP = 90/60 mm Hg, HR 145 Beats/min, cardiac output...

    Case 1 – New data set: ABP = 90/60 mm Hg, HR 145 Beats/min, cardiac output = 3 l/min, CVP = 0 mm Hg, PAP = 30/10 mm Hg, PCWP = 2 mm Hg. Calculate the SV and fill in the table below and answer the question below. Mean arterial pressure (MAP) mmHg Systemic vascular resistance (SVR) dynes Mean pulmonary artery pressure (PA mean) mmHg Pulmonary vascular resistance (PVR) dynes Stroke volume (SV) ml Regarding Case 1. The cardiac output...

  • Case 1 – New data set: ABP = 90/60 mm Hg, HR 145 Beats/min, cardiac output...

    Case 1 – New data set: ABP = 90/60 mm Hg, HR 145 Beats/min, cardiac output = 3 l/min, CVP = 0 mm Hg, PAP = 30/10 mm Hg, PCWP = 2 mm Hg. Calculate the SV and fill in the table below and answer the question below. Mean arterial pressure (MAP) mmHg Systemic vascular resistance (SVR) dynes Mean pulmonary artery pressure (PA mean) mmHg Pulmonary vascular resistance (PVR) dynes Stroke volume (SV) ml Regarding Case 1. The best treatment...

  • Case 1- New data set: ABP: 90/60 mm Hg, HR 145 beats/min, cardiac output=3 l/min, CVP=0...

    Case 1- New data set: ABP: 90/60 mm Hg, HR 145 beats/min, cardiac output=3 l/min, CVP=0 mm Hg, PAP=30/10 mm Hg, PCWP=2 mm Hg. I calculated that MAP=70 mm Hg, SVR=1866.7 dynes, PA mea=16.7 mm HG, PVR=139.2 dynes, and SV=20.1 ml. I just need help with the conceptual questions: A. How would SV normally respond if venous return increases? (increase, decrease, or nothing) B. What would be the best treatment for the patient? (diuretics, aortic balloon pump, intravenous fluid admin,...

  • Matching (put ONE correct letter next to the term in first column). I. pathology underlying why...

    Matching (put ONE correct letter next to the term in first column). I. pathology underlying why diuretics (meds that increase urination) are given in a person with heart failure A. decreased cardiac output kidneys sense this and increase renin as a compensatory response RAAS kicks in but its results exacerbate the problem. B. increased preload is almost always part of the heart failure picture II. mini-concept map that explains S&S of LHF III. person has chronic lung diseaselungs very stiff-...

  • Please provide detailed explanation for each answer. Thanks Case Study: You are a nurse working in...

    Please provide detailed explanation for each answer. Thanks Case Study: You are a nurse working in an emergency department. Your new admission is a 22 year-old-male brought in by the paramedics after fainting during a try out session for his university's competitive hockey team. The patient states he was experiencing some chest pain and dizziness right before fainting. He denies having any past medical history or taking any medication. His vital signs at the time the paramedics arrived at the...

ADVERTISEMENT
Free Homework Help App
Download From Google Play
Scan Your Homework
to Get Instant Free Answers
Need Online Homework Help?
Ask a Question
Get Answers For Free
Most questions answered within 3 hours.
ADVERTISEMENT
ADVERTISEMENT
ADVERTISEMENT